Chest and Abdominal Wall Reconstruction Flashcards

1
Q

An otherwise healthy 47-year-old man comes to the office because of the recurrent ventral hernia shown. He does not smoke cigarettes. Physical examination shows “Swiss cheese–type” defects, which are confirmed by CT scan. He desires reconstruction. Which of the following is the most appropriate surgical treatment?
A) Component separation with mesh placement
B) Laparoscopic hernia repair with mesh placement
C) Open hernia reduction with bridging mesh placement
D) Open reduction with free tensor fascia lata flap
E) Total autologous component separation

A

A) Component separation with mesh placement

Given the size and history of the defect, the patient described has a significant chance of recurrence of his hernia. To optimize the chances of a functional recovery that is durable and has the lowest chance of recurrence, the component separation technique with mesh reinforcement (synthetic or biologic) is indicated.

How well did you know this?
1
Not at all
2
3
4
5
Perfectly
2
Q

Component separation technique overview

A

The component separation technique, originally described by Ramirez et al. in 1990, allows for recruitment of the rectus complex towards the midline to facilitate primary reapproximation (in the best-case scenario) or at least decrease the size of the residual defect.

Component separation allows for reduction of the hernia defect size after hernia reduction, and leads to lower recurrence rates versus hernia reduction and bridging mesh repair alone.

How well did you know this?
1
Not at all
2
3
4
5
Perfectly
3
Q

Mesh reinforcement: Overlay vs underlay

A

Studies have shown that reinforcement of hernia repairs with mesh decreases recurrence rates by 50 to 75%, even in secondary repairs. It is not clear, however, whether the mesh should be placed as an overlay or underlay. Nonetheless, mesh reinforcement leads to lower recurrence rates, especially if combined with component separation.

How well did you know this?
1
Not at all
2
3
4
5
Perfectly
4
Q

Tensor fascia lata flap for hernia repairs

A

A tensor fascia lata flap, originally described for large hernia repairs because it allows transfer of fascia to the abdomen, does not demonstrate superior results compared to available meshes. It also incurs donor site morbidity versus the mesh reinforcement techniques

How well did you know this?
1
Not at all
2
3
4
5
Perfectly
5
Q

A 24-year-old woman is scheduled to undergo correction of the defect shown in the photographs. Physical examination shows an elevated inframammary fold and herniation of breast tissue through the areolar complex. Which of the following is the most appropriate operative plan?
A) Latissimus dorsi musculocutaneous flaps and placement of silicone gel prostheses
B) Subfascial placement of silicone gel prostheses with mastopexy
C) Subglandular placement of silicone gel prostheses
D) Submuscular placement of silicone gel prostheses
E) Submuscular placement of silicone gel prostheses with mastopexy and scoring of the gland

A

E) Submuscular placement of silicone gel prostheses with mastopexy and scoring of the gland

How well did you know this?
1
Not at all
2
3
4
5
Perfectly
6
Q

Tuberus breast deformity

A

Tuberous breast deformity is a rare congenital condition that results in aberrant breast shape due to a constricting ring at the breast base. Abnormal development results in breast tissue deficiency, herniation of breast tissue into the nipple-areola complex, areolar enlargement, and breast asymmetry

How well did you know this?
1
Not at all
2
3
4
5
Perfectly
7
Q

Surgical correction of tuberous breast deformity

A

Surgical correction is challenging, but it can be achieved in a single-stage operation. This procedure should include submuscular placement of silicone or saline prostheses, mastopexy with areolar reduction, and scoring of the gland to relieve the constricting tissue.

How well did you know this?
1
Not at all
2
3
4
5
Perfectly
8
Q

An 18-year-old woman comes to the office because of a small area of titanium extruding through the skin overlying the sacrum 12 months after undergoing a successful posterior spinal fusion and an autologous bone graft for myelodysplasia and lumbosacral kyphotic deformity. Physical examination shows a midline scar with a 2 × 2-cm wound at the level of the fourth lumbar vertebra with a visible screw. Which of the following is the most appropriate next stepin management?
A) Bilateral skin advancement flaps
B) Free rectus abdominis muscle flap
C) Gluteus maximus muscle flap
D) Negative pressure wound therapy and skin graft
E) Removal of the hardware

A

E) Removal of the hardware

The most appropriate next step in management of the exposed titanium isremoval of the hardware. The titanium rod or one of the screws likely loosened and subsequently migrated to extrude through the skin. The vertebral column is adequately fused 1 year following the procedure.

Although local skin flaps, skin grafts, regional muscle transfer, and free flaps will provide vascularized soft-tissue coverage over the exposed hardware, these options ultimately will fail. The exposed hardware is much stronger than any soft-tissue coverage and will extrude through a flap, especially when the patient is supine.

How well did you know this?
1
Not at all
2
3
4
5
Perfectly
9
Q

The vertebral column is adequately fused ________ following autologous bone graft posterior spinal fusion

A

1 year

How well did you know this?
1
Not at all
2
3
4
5
Perfectly
10
Q

A 45-year-old woman with rectal cancer is scheduled to undergo abdominoperineal resection and posterior vaginectomy following neoadjuvant chemotherapy and radiation therapy. Which of the following is the most appropriate method of reconstruction of the posterior vaginal defect?
A) Free anterolateral thigh fasciocutaneous flap
B) Primary repair of the perineal and vaginal defects
C) Pudendal thigh (Singapore) flap
D) Skin grafting
E) Vertical rectus abdominis myocutaneous flap

A

E) Vertical rectus abdominis myocutaneous flap

Although the pedicled ALT flap has recently been described for vaginoperineal defects, the VRAM flap remains the gold standard. Primary repair, even if technically possible, would be associated with unacceptable risk of wound-healing problems.

How well did you know this?
1
Not at all
2
3
4
5
Perfectly
11
Q

Pros of a vertical rectus abdominis myocutaneous flap

A

The vertical rectus abdominis myocutaneous (VRAM): ease of harvest, minimal donor-site morbidity, large skin paddle (which can resurface both the perineum and the vagina), reliable blood supply, and favorable arc of rotation (for vaginoperineal defects)

How well did you know this?
1
Not at all
2
3
4
5
Perfectly
12
Q

Gold standard for vaginoperineal defects

A

Although the pedicled ALT flap has recently been described for vaginoperineal defects, the VRAM flap remains the gold standard.

How well did you know this?
1
Not at all
2
3
4
5
Perfectly
13
Q
A 46-year-old woman undergoes breast reconstruction with a pedicled transverse rectus abdominis musculocutaneous (TRAM) flap. Weight is 185 lb (84 kg); BMI is 32 kg/m2. The donor site is reconstructed with mesh. Six months postoperatively, the patient develops a bulge at the donor site. Which of the following best represents the deepest layer of the anterior rectus sheath when it is harvested caudal to the arcuate line?
A) External oblique aponeurosis
B) Internal oblique aponeurosis
C) Rectus abdominis fascia
D) Transversalis fascia
E) Transversus abdominis aponeurosis
A

E) Transversus abdominis aponeurosis

How well did you know this?
1
Not at all
2
3
4
5
Perfectly
14
Q

Rectus sheath

A

The rectus sheath is the semifibrous compartment that encompasses the rectus abdominis muscle and consists of an anterior and posterior sheath created by the aponeurotic extensions of the external oblique, internal oblique, and transversus abdominis muscles.

How well did you know this?
1
Not at all
2
3
4
5
Perfectly
15
Q

Rectus sheath proximal to the arcuate line

A

Proximal to the arcuate line, which is located approximately at the level of the anterior superior iliac spine, the anterior rectus sheath comprises the aponeuroses of the external oblique and the anterior leaf of the internal oblique. The internal oblique has two leaves, the deeper of which contributes to the posterior rectus sheath along with the transversus abdominis and transversalis fascia.

How well did you know this?
1
Not at all
2
3
4
5
Perfectly
16
Q

Rectus sheath at the level of the arcuate line

A

At the level of the arcuate line, the posterior leaf of the internal oblique aponeurosis and the transversus abdominis aponeurosis travel superficially to the rectus abdominis, making the posterior sheath very weak. When the anterior rectus sheath is defective at this level, the likelihood for bulging or hernia is high.

The level of the arcuate line is not always clear from topographical landmarks, which can make the planning of a transverse rectus abdominismusculocutaneous (TRAM) flap more difficult and increase the risk for a hernia complication

How well did you know this?
1
Not at all
2
3
4
5
Perfectly
17
Q

An otherwise healthy 25-year-old man comes to the office because of a 10-year history of bilateral gynecomastia. Physical examination shows breast enlargement with skin redundancy and palpable glandular and fatty tissue. Which of the following is the most appropriate next step in management?
A) Determination of 17-ketosteroid level in urine
B) Mammography
C) Referral to an endocrinologist
D) Surgical excision
E) Testicular ultrasound

A

D) Surgical excision

The most appropriate next step for the patient described, who is young with a long history of bilateral gynecomastia since puberty, is surgical excision. The onset of gynecomastia correlates with transient elevations of plasma estradiols prior to the completion of puberty so that the androgen-to-estrogen ratio is altered.

How well did you know this?
1
Not at all
2
3
4
5
Perfectly
18
Q

Most cases of gynecomastia present when?

A

Most cases of gynecomastia present at puberty, with an incidence as high as 65% in boys 14 to 15 years of age. The condition disappears during the late teens, with only 7.7% remaining at age 17 years. The incidence rises again with progressive age.

How well did you know this?
1
Not at all
2
3
4
5
Perfectly
19
Q

Severity/concern when gynecomastia is found

A

The condition is often a normal finding, even though it may be associated with a more serious disease in occasional cases

How well did you know this?
1
Not at all
2
3
4
5
Perfectly
20
Q

Systemic causes of gynecomastia

A

In certain cases, systemic causes —such as liver disease, lung carcinoma, testicular carcinoma, adrenal tumors, thyroid disease, testosterone imbalance, and Klinefelter syndrome —or drugs like marijuana, should be considered. For instance, a prepubescent boy presenting with gynecomastia would cause concern. Another example would be if an adult man presented with a 6-month history of unilateral gynecomastia. A work-up including liver function test, urine studies, testicular examination, endocrinology evaluation, and possibly mammography should be done if there is a possibility of cancer (ie, patients with Klinefelter syndrome).

How well did you know this?
1
Not at all
2
3
4
5
Perfectly
21
Q
A 17-year-old girl comes to the office for consultation regarding augmentation mammaplasty. Physical examination shows right-sided hypoplasia of the chest wall and breast. Absence of the pectoralis major muscle is noted. This patient is most likely to have which of the following hand anomalies?
A) Micromelia
B) Oligodactyly
C) Phocomelia
D) Polydactyly
E) Syndactyly
A

E) Syndactyly

How well did you know this?
1
Not at all
2
3
4
5
Perfectly
22
Q

A 55-year-old man comes to the office because of the ventral hernia shown. History includes a sigmoid colectomy for diverticular disease using a midline incision 7 years ago. Postoperatively, an incisional hernia developed. He underwent mesh repair, which was unsuccessful. Physical examination shows a 28 (horizontal at the level of the umbilicus) by —25-cm (vertical) palpable fascial defect. Which of the following is the most appropriate method of reconstruction of the abdominal wall?
A ) Component separation technique only
B ) Component separation technique with mesh repair
C ) Mesh repair only
D ) Tissue expansion and component separation technique with mesh repair
E ) Use of a pedicled tensor fascia lata flap

A

B ) Component separation technique with mesh repair

A component separation repair alone would be insufficient because the defect described is 28 cm wide. There would still be a persistent defect in the fascia. Therefore, component separation with supplemental mesh would be required to adequately close this defect. Component separation mobilizes functional muscle medially to help restore abdominal wall integrity and minimizes the amount of bridging that would be repaired with mesh.

Tissue expansion can be useful in situations where skin coverage may be lacking. Tissue expansion is unnecessary in the patient described because skin coverage is not an issue.

How well did you know this?
1
Not at all
2
3
4
5
Perfectly
23
Q

Components separation overview

A

Originally described by Ramirez et al. in 1990, the technique involves incising the external oblique fascia lateral to the rectus muscle. The external oblique is elevated off of the internal oblique muscle in a loose areolar plane that is largely avascular, except for the intercostal branches to the external oblique muscles that are located far posteriorly. The posterior rectus sheath can be incised longitudinally to provide an additional 1 to 2-cm mobilization if needed. The attachments of the internal oblique to the rectus muscle are preserved, as are the intercostal neurovascular bundles that supply the rectus muscle.

How well did you know this?
1
Not at all
2
3
4
5
Perfectly
24
Q

Function of the rectus muscle after components separation

A

The rectus muscle is kept innervated and functional.

How well did you know this?
1
Not at all
2
3
4
5
Perfectly
25
Q

How much can be mobilized in a components separation

A

Under ideal conditions, mobilization of the rectus-internal oblique muscle flap on both sides allows for closure of defects at the epigastrium, mid abdomen, and low abdomen of 10, 20, and 6 cm, respectively.

The posterior rectus sheath can be incised longitudinally to provide an additional 1 to 2-cm mobilization if needed.

How well did you know this?
1
Not at all
2
3
4
5
Perfectly
26
Q

Larger abdominal defects repaired with mesh alone can be associated with:

A

A mesh repair alone would close the fascial defect but would do nothing to restore the functionality of the abdominal wall. Large defects repaired with mesh can be associated with hernia recurrence and long-term bulging.

How well did you know this?
1
Not at all
2
3
4
5
Perfectly
27
Q
A 30-year-old woman is referred for consultation because of a 3-year history of pain and irritation of the labia minora when she exercises. She now has enlarged, protuberant labia. Labioplasty is planned. Which of the following postoperative adverse outcomes is most likely in this patient?
A ) Group B streptococcal infection
B ) Incomplete correction
C ) Keloid formation
D ) Painful intercourse
A

B ) Incomplete correction

How well did you know this?
1
Not at all
2
3
4
5
Perfectly
28
Q

Indications for labioplasty

A

Labioplasty is a common surgical procedure used to reduce the size of excessively protruding or enlarged labia. The condition can result in discomfort or irritation with exercise or intercourse. Frequently, surgery is sought for cosmetic reasons alone.

How well did you know this?
1
Not at all
2
3
4
5
Perfectly
29
Q

Goal of labioplasty

A

The goal of surgery is to reduce the size of the labia so that they become closer to the level of, or posterior to, the labia majora.

How well did you know this?
1
Not at all
2
3
4
5
Perfectly
30
Q

Ideal scar positioning for labioplasty

A

Direct amputation-excision should be avoided, as an anterior scar line can cause postoperative discomfort in clothing or with sexual activity. Effective surgical methods use incisions that place the scars inferior, interior, or transverse, such as the inferior wedge resection with superior flap technique.

How well did you know this?
1
Not at all
2
3
4
5
Perfectly
31
Q

Most common complaint after labioplasty

A

The most frequent postoperative complaint is incomplete correction, wherein the patients are not completely satisfied with the aesthetic result (about 15% of cases).

How well did you know this?
1
Not at all
2
3
4
5
Perfectly
32
Q
A 56-year-old woman who has type 2 diabetes mellitus is scheduled to undergo removal of prosthetic mesh that became infected after ventral hernia repair. Weight is 311 lb (141 kg); BMI is 53 kg/m2. Reconstruction with component separation technique and onlay grafting of reinforcing mesh is planned. Compared with polytetrafluoroethylene, acellular dermal matrix is preferable for this patient because of its ability to undergo which of the following processes?
A ) Contraction 
B ) Encapsulation 
C ) Imbibition 
D ) Inosculation 
E ) Revascularization
A

E ) Revascularization

How well did you know this?
1
Not at all
2
3
4
5
Perfectly
33
Q

What is most likely the key to ADM’s successful application in contaminated scenarios for abdominal hernia repair?

A

The ability of acellular dermal matrix (ADM) to revascularize is most likely the key to its successful application in contaminated scenarios. Revascularization subsequently helps clear bacteria

How well did you know this?
1
Not at all
2
3
4
5
Perfectly
34
Q

A 41-year-old man is being evaluated prior to ventral hernia repair. History includes a traumatic abdominal injury with exposed bowel 4 years ago treated with negative pressure dressings and skin grafting. A separation of components technique will be used. The connection between which of the following layers will most likely remain intact during this procedure?
A ) Bowel and fascial edge
B ) External oblique and internal oblique
C ) External oblique and subcutaneous layer
D ) Internal oblique and transversus abdominis
E ) Rectus muscle and posterior rectus sheath

A

D ) Internal oblique and transversus abdominis

How well did you know this?
1
Not at all
2
3
4
5
Perfectly
35
Q

During a components separation, why is the internal oblique not usually separated from the transversus abdominis muscle?

A

The internal oblique muscle is usually not separated from the transversus abdominis muscle because it contains the intercostal nerves and blood vessels. This makes dissection difficult, bloody, and heightens the risk of denervating the rectus abdominis muscle.

How well did you know this?
1
Not at all
2
3
4
5
Perfectly
36
Q

A 13-year-old boy is brought for evaluation by his mother because of development of his breasts during the past 3 months. The patient is at the 50th percentile for height and weight; BMI is 22 kg/m2. Physical examination shows no abnormalities except for moderately severe bilateral symmetric gynecomastia. Which of the following is the most appropriate next step in management?
A ) Liposuction
B ) Liver function studies
C ) Observation and follow-up in 12 months
D ) Surgical excision
E ) Testicular ultrasonography

A

C ) Observation and follow-up in 12 months

How well did you know this?
1
Not at all
2
3
4
5
Perfectly
37
Q

How long should you wait before considering surgery for gynecomastia?

A

Rohrich advocates waiting 12 months prior to considering surgery in the absence of other findings.

How well did you know this?
1
Not at all
2
3
4
5
Perfectly
38
Q

A 36-year-old man is scheduled to undergo correction of grade IV gynecomastia. On physical examination, the distance from the nipple to the sternal notch is 25 cm and the inframammary crease is 4 cm inferior to the lower pectoral border. A mastectomy and free nipple grafting are planned. Which of the following is the most appropriate configuration and position of the new nipple-areola complex?
A ) Oval; placement at the fourth-to-fifth intercostal space
B ) Oval; placement 20 cm from the mid clavicle
C ) Oval; placement 25 cm from the sternal notch
D ) Round; placement 25 cm from the sternal notch
E ) Round; placement at the fourth-to-fifth intercostal space

A

A ) Oval; placement at the fourth-to-fifth intercostal space

How well did you know this?
1
Not at all
2
3
4
5
Perfectly
39
Q

Average size/shape of the male areolar/nipple complex

A

One consistent finding with respect to nipple-areola configuration in men is the shape. In one study of 100 healthy male subjects performed in 2001, it was found that 91% had an oval-shaped complex with a mean ratio of horizontal-to-vertical diameter of 27:20 mm. The mean nipple-areola diameter of the 9% of patients who had a round complex was 2.3 cm

How well did you know this?
1
Not at all
2
3
4
5
Perfectly
40
Q

Average position of the nipple/areolar complex in men

A

It has been found that the center of the nipple-areola complex was in the fourth intercostal space in 75% of patients and in the fifth intercostal space in 23% of patients

How well did you know this?
1
Not at all
2
3
4
5
Perfectly
41
Q

A 45-year-old man is scheduled to undergo abdominal wall reconstruction to repair a central defect resulting from abdominal compartment syndrome. Removal of a temporary colostomy placed two years ago and an autologous repair usinga component separation technique are planned. Which of the following techniques is LEAST likely to preserve innervation of the rectus muscles in this patient?
A ) Endoscopic release of the external oblique aponeurosis
B ) Laparoscopic division of the transversalis muscle
C ) Periumbilical perforator sparing components release
D ) Use of an anterior sheath fascial turnover flap
E ) Use of a rectus muscle medial turnover flap

A

E ) Use of a rectus muscle medial turnover flap

How well did you know this?
1
Not at all
2
3
4
5
Perfectly
42
Q

Components separation: harvesting the rectus muscle as a turnover flap

A

Harvesting the rectus muscle as a turnover flap will provide additional vascularized tissue medially, but this maneuver divides the intercostal nerves that travel between the internal oblique and transversalis muscle, providing motor supply to the rectus. The denervated muscle loses its previous advantage as an innervated, functional flap.

How well did you know this?
1
Not at all
2
3
4
5
Perfectly
43
Q

Components separation: modifications that can provide further advancement

A

Important modifications to this technique, which can provide up to 10 cmof advancement per side, include preservation of the periumbilical perforators, endoscopic release of the external oblique, laparoscopic release of the transversalis only, and anterior sheath fascial flap to provide additional midline reach.

How well did you know this?
1
Not at all
2
3
4
5
Perfectly
44
Q

A 25-year-old man is brought to the emergency department after sustaining injuries during a motorcycle collision. He is alert, cooperative, and vital signs are stable. Examination shows a large skin avulsion with deep lacerations of the left chest and back. Radiograph of the chest shows fractures of multiple ribs; no other abnormalities are noted. During irrigation of the chest wounds, his breathing becomes labored and oxygen saturation and blood pressure decrease precipitously. After stabilizing the airway, which of the following is the most appropriate next step in management?
A ) Administration of a vasopressor
B ) Diagnostic peritoneal lavage
C ) Needle decompression
D ) Placement of central and arterial catheters
E ) Repeat radiograph of the chest

A

C ) Needle decompression

How well did you know this?
1
Not at all
2
3
4
5
Perfectly
45
Q

The most likely diagnosis for a precipitous drop in oxygenation and blood pressure in the setting of a large open wound of the chest is:

A

The most likely diagnosis for a precipitous drop in oxygenation and blood pressure in the setting of a large open wound of the chest is tension pneumothorax.

How well did you know this?
1
Not at all
2
3
4
5
Perfectly
46
Q

Location for needle decompression of tension pneumothorax

A

Needle decompression with alarge-bore intravenous catheter is performed in the second intercostal space in the midclavicular line.

How well did you know this?
1
Not at all
2
3
4
5
Perfectly
47
Q

Treatment of tension pneumothorax

A

Needle decompression with alarge-bore intravenous catheter is performed in the second intercostal space in the midclavicular line. This procedure is followed by tube thoracostomy for stable decompression of the thoracic cavity, thereby allowing lung inflation

How well did you know this?
1
Not at all
2
3
4
5
Perfectly
48
Q
A 4-month-old female infant is brought to the office because of redness and irritation of her navel, along with occasional drainage of a small amount of fluid. Examination of the umbilicus shows a pinpoint epithelialized tract. Which of the following is the most likely diagnosis?
A ) Omphalitis
B ) Umbilical granuloma
C ) Umbilical hernia
D ) Urachal cyst
E ) Urachal sinus
A

E ) Urachal sinus

How well did you know this?
1
Not at all
2
3
4
5
Perfectly
49
Q

Urachal sinus

A

A urachal sinus is derived from a persistently patent urachus with a sinus that drains to the umbilicus. Intermittent drainage is often the result of infections from this sinus tract. Surgical excision is recommended to avoid future infections and potential malignant degeneration.

How well did you know this?
1
Not at all
2
3
4
5
Perfectly
50
Q

Omphalitis

A

Omphalitis is an infection of the umbilical stump that presents as cellulitis. It is rare in industrialized countries but can be a common cause of neonatal mortality in developing areas. The source of infection is polymicrobial.

How well did you know this?
1
Not at all
2
3
4
5
Perfectly
51
Q

Umbilical granuloma

A

An umbilical granuloma or pyogenic granuloma is a benign inflammatory reaction to a healing umbilicus. This bright red, friable lesion does not have a sinus tract or drainage of fluid

52
Q

Urachal cyst

A

An urachal cyst is a fluid-filled structure that is also a remnant of a patent urachus; however, the two ends (umbilicus and bladder) are obliterated. They are rarely symptomatic in early childhood; however, in adolescence they can present as an abdominal mass and cause pain. These cysts can also develop infections as well.

53
Q
A neonate with a small nasal hemangioma is examined and found to have supernumerary nipples. Family history includes polythelia in the patient’s mother and other maternal family members. The patient and family members are at increased risk for which of the following abnormalities?
A ) Ear anomalies
B ) Renal anomalies
C ) Situs inversus
D ) Submucous clefts
E ) Thyroglossal duct cysts
A

B ) Renal anomalies

54
Q

Polythelia

A

Supranumary nipples

55
Q

Associations with polythelia

A

A strong association exists between renal disease and polythelia. Kidney abnormalities such as cysts, duplications, or unilateral renal agenesis can be found in 14% of sporadic and 32% of familial cases compared with those seen in 1% to 2% of the general population.

56
Q

A 9-year-old boy is referred for evaluation because his breasts have been enlarging during the past six months without pain. Tanner stage is 1. Height and weight are in the 50th percentile. He takes no medications, and his medical history is noncontributory. Which of the following is the most appropriate next step in establishing the diagnosis?
A ) Abdominal CT and renal ultrasonography
B ) Karyotyping
C ) Measurement of serum gonadotropin concentrations
D ) Testicular examination and ultrasonography
E ) No further testing is necessary

A

D ) Testicular examination and ultrasonography

The patient described comes to the office with early breast enlargement without other signs of pubertal development. Prepubertal gynecomastia is an uncommon condition, which should prompt a thorough workup for a possible pathologic cause.

Mammogram and/or ultrasonography of the breast are reasonable choices if history and physical examination are suggestive of breast cancer. Breast cancers usually present as a hard, unilateral mass in an older patient. Karyotype testing would be warranted to rule out Klinefelter syndrome if feminizing characteristics are found on physical examination.

57
Q

Preferred screening method for evaluating patients with idiopathic gynecomastia

A

Ultrasonography is the preferred screening method, and is cheaper and more accurate than hormone assays.

58
Q
Which of the following pharmacologic agents has NOT been implicated as a cause of gynecomastia?
A ) Cannabis sativa
B ) Chlorpromazine
C ) Isoniazid
D ) Metformin
E ) Spironolactone
A

D ) Metformin

59
Q

Agents that can cause gynecomastia

A
Therapy w/ estrogen or androgen
Stilbestrol
Antiandrogens
Chorionic gonadotropin
Sprironolactone
Reserpine
Phenothiazines
Methyldopa
Meprobamate
Ergotamine
Diazepam
Hydantoin
Marijuana
Digitalis
Isoniazid
Anabolic steroids
Chorpromazine
60
Q
Which of the following best describes the benefit of using acellular human dermal matrix compared with alloplastic mesh in complex abdominal wall reconstruction?
(A)Adhesion potential
(B)Cost effectiveness
(C)Definitive scar formation
(D)Rapid absorption
(E)Resistance to infection
A

(E)Resistance to infection

Because of retainedvascular channels, this material revascularizes rapidly and resists infection.

61
Q

Eventual strength of allured (abdominal wall recon)

A

The matrix serves as a soft-tissue scaffold, retaining its original strength and becoming incorporated with minimal scar or adhesion formation.

62
Q
A 20-year-old man is scheduled for gynecomastia reduction by the use of ultrasound-assisted lipoplasty. In planning this procedure, which of the following liposuction techniques has the highest risk of thermal injury to the skin?
(A)Dry
(B)Wet
(C)Superwet
(D)Tumescent
A

(A)Dry

63
Q

Ultrasound assisted liposuction and reducing risk of thermal injury

A

Ultrasound-assisted liposuction in a dry environment increases the risk of thermal injury and overlying skin necrosis. The introduction of subcutaneous fluid in the wet, superwet, and tumescent techniques helps cool the probe and decreases the risk of injury.

Additionally, avoidance of end hits, strict continuous movement of the probe, continuous cold saline irrigation, and use of a probe sheath and a wet towel as a skin guard also protect against thermal injury when ultrasound energy is delivered to the tissues.

64
Q
A 17-year-old girl comes to the office for consultation regarding breast asymmetry. She wears a size 34B brassiere, which fits the right breast but is too large for the left breast. On examination of the left side of the chest, there is no palpable breast parenchyma and no nipple-areola complex. No abnormalities of the muscles and ribs are noted. Development of the right breast is Tanner stage 4. Which of the following is the most likely diagnosis?
(A)Amastia
(B)Amazia
(C)Athelia
(D)Pectus excavatum
(E)Poland syndrome
A

(A)Amastia

65
Q

Congenital absence of the nipple-areola complex and glandular tissue

A

Amastia: a congenital condition that involves absence of the nipple-areola complex and glandular tissue.

66
Q

Congenital absence of breast glandular tissue only

A

Amazia

67
Q

Congenital absence of nipple alone

A

Athelia

68
Q

Pectus excavatum

A

Pectus excavatum is a congenital defect of the thoracic wall, consisting of a concave deformity of the mid thorax. Reconstruction is indicated for aesthetic reasons or, in severe cases, for relief of cardiorespiratory dysfunction

69
Q

Which of the following findings is most indicative of Poland syndrome?
(A)Abnormalities of the chest wall
(B)Absence of the nipple
(C)Absence of the sternal head of the pectoralis major muscle
(D)Brachysyndactyly
(E)Hypoplasia of the latissimus dorsi muscle

A

(C)Absence of the sternal head of the pectoralis major muscle

70
Q

Criteria for definitive diagnosis of Poland syndrome

A

For the definitive diagnosis of Poland syndrome, patients must have absence or hypoplasia of the sternal head of the pectoralis major muscle on the affected side.

71
Q
Accessory mammary structures are most frequently found at which of the following sites?
(A)Axilla
(B)Buttock
(C)Neck
(D)Thigh
(E)Vulva
A

(A)Axilla

72
Q

Where are accessory mammary structures found?

A

Accessory mammary structures are found along the embryonic milk line, which forms on the ventrolateral body wall from the axilla to the groin. These structures include most supernumerary breasts, which are most often found in the axilla just above or belowthe normal breast or in the groin.

73
Q

A 14-year-old girl is brought to the office for consultation regarding breast asymmetry. She is anxious and embarrassed about the appearance of her right breast. Physical examination shows hypoplasia of the right breast with superior nipple displacement, absence of the sternocostal head of the pectoralis muscle, and deficiency of subcutaneous fat and axillary hair. The left breast is B-cup size and appears normal. Which of the following is the most appropriate initial step for reconstruction of the right breast?
(A)Implantation of a saline breast prosthesis covered by a latissimus dorsi flap
(B)Injection of autologous fat
(C)Placement of an adjustable subcutaneous tissue expander
(D)Placement of an adjustable tissue expander covered by a latissimus dorsi flap
(E)Reconstruction with a TRAM flap

A

(C)Placement of an adjustable subcutaneous tissue expander

In the patient described, a subcutaneous tissue expander can be placed because the sternal portion of the pectoralis is missing.

Placement of a saline breast prosthesis covered by a latissimus dorsi flap is not appropriate because additional development will likely require a larger prosthesis. Furthermore, as the patient grows, the latissimus with fat added or buried skin paddle may be required to further enhance areas of soft-tissue deficiency

74
Q

TRAM flaps and childbirth

A

contraindicated prior to childbirth

75
Q
A 65-year-old man who underwent three-vessel coronary artery bypass grafting (CABG) five weeks ago comes to the office because he has a draining lesion near the sternotomy incision. The CABG procedure included harvest of the patient’s left internal mammary artery. Temperature is 38.7EC (101.7EF). Physical examination shows a 3-mm papule at the manubrium of the healed sternotomy incision. CT scan of the chest shows a sinus tract leading to the internal sternal plate. In addition to removal of sternal wires and debridement of the wound, which of the following is the most appropriate management?
(A)Continuous irrigation
(B)Negative-pressure wound therapy
(C)Omental flap
(D)Pectoralis major muscle flaps
(E)Rectus abdominis muscle flap
A

(D)Pectoralis major muscle flaps

The patient described had a Type III infection with a deep sinus tract as confirmed by CT scan.

A single-stage approach with bilateral pectoralis major muscle flaps results in quicker recovery, improved cosmesis, and decreased morbidity and mortality compared with staged approaches. Given the chronicity of the wound, all foreign bodies, including retained suture material and wires, should be removed.
Pectoralis major turnover flaps provide additional bulk and might therefore be considered preferable. However, the left internal mammary arteryhas been harvested in this patient, compromising the blood supply to the left pectoralis major turnover flap.

76
Q

Type I post-sternotomy infection

A

Type I infections occur within the first week after sternotomy and have no bony involvement.

77
Q

Type II post-sternotomy infection

A

Type II infections occur during the second to fourth weeks after sternotomy. Bony involvement is frequent, but chostochondritis is rare.

78
Q

Type III post-sternotomy infection

A

Type III infections occur months to years after sternotomy and typically involve chronic draining sinus tracts. Osteomyelitis, chostochondritis, and retained foreign bodies are all common

79
Q

Treatment of Type III post-sternotomy infection

A

A single-stage approach with bilateral pectoralis major muscle flaps results in quicker recovery, improved cosmesis, and decreased morbidity and mortality compared with staged approaches. Given the chronicity of the wound, all foreign bodies, including retained suture material and wires, should be removed.

80
Q
A 57-year-old man comes to the office for consultation regarding enlargement of the breasts. Physical examination shows bilateral large, ptotic, female-appearing breasts with firm, tender, glandular-like tissue deep to each nipple. Laboratory studies show increased beta-human chorionic gonadotropin level. Which of the following studies is the most appropriate next step in establishing the diagnosis?
(A) Biopsy of the breast
(B) CT scan of the abdomen
(C) Mammography
(D) MRI of the brain
(E) Ultrasonography of the testes
A

(E) Ultrasonography of the testes

81
Q

When is mammography indicated for gynecomastia?

A

Mammography is not routinely used unless there is a finding on physical examination that shows possible presence of breast cancer; there is no known association between gynecomastia and breast cancer (except in Klinefelter syndrome). I

82
Q

When is brain imaging indicated for gynecomastia?

A

Imaging of the brain is not routinely ordered unless there is some other finding suggestive of a brain tumor.

83
Q

Workup for gynecomastia with increased B-hcg

A

If the B-hCG concentration is increased, ultrasonography of the testes is indicated to rule out germ cell and non–germ cell tumors. CT scan of the abdomen should be ordered only if ultrasonography of the testes is negative

84
Q
A 63-year-old man comes to the office for consultation regarding correction of large breasts. Detailed history and physical examination show no cause of his condition. The breasts are a size C cup with ptosis and excessive skin. The nipples are 5 cm in diameter. Mastectomy and free nipple grafts are planned. The desired new size and shape of each areola are closest to which of the following?
(A) 1 cm, round
(B) 2 cm, round
(C) 2 cm, oval
(D) 3 cm, oval
(E) 4 cm, round
A

(D) 3 cm, oval

85
Q

A 5-year-old girl is brought to the office by her parents for consultation regarding deformity of the left side of the chest and the left upper extremity. Physical examination shows athelia, brachydactyly, and absence of the left pectoralis major muscle. Which of the following additional findings is most likely?
(A) Absence of the 12th rib
(B) Bilateral syndactyly
(C) Hypoplasia of the left rectus abdominis muscle
(D) Left brachial cyst
(E) Shortened right ulna

A

(A) Absence of the 12th rib

The child described has Poland syndrome, which is characterized by anomalies of the upper limbs and torso. The most common finding in patients with Poland syndrome is absence of the sternal head of the pectoralis major muscle. Other muscles, including the external oblique, latissimus dorsi, pectoralis minor, and serratus muscles, may be absent or hypoplastic. The rectus abdominis muscle is not involved. Ribs and costal cartilages of the affected side may be absent. Other anomalies of the chest include athelia and hypoplastic skin covering the chest wall. Anomalies of the ipsilateral upper extremity include brachysyndactyly as well as foreshortening and hypoplasia of the forearm.

86
Q

Where are, in general, deformities in Poland syndrome located versus the absent sternal head of pectorals major?

A

Ipsilateral

87
Q

A 27-year-old woman is evaluated for a recurrent abdominal desmoid tumor. CT scan shows a mass that occupies the full-thickness right musculofascial abdominal wall, involving the rectus abdominis muscle and oblique muscles, including lateral to the semilunar line. Resection is performed. Photographs of the defect are shown. Which of the following is the most appropriate management?

A) Bilateral component separation, primary skin closure with incisional topical negative pressure wound therapy, adjuvant chemotherapy
B) Left component separation, bridging wide intraperitoneal underlay biologic mesh, primary closure skin
C) Pedicled right anterolateral thigh flap with rectus femoris, no mesh
D) Placement of a bridging inlay of biologic mesh with primary split-thickness skin grafting and negative pressure wound therapy
E) Placement of a bridging inlay of uncoated heavyweight polypropylene mesh with adjuvant radiation therapy

A

B) Left component separation, bridging wide intraperitoneal underlay biologic mesh, primary closure skin

The lesion in this patient is a recurrent desmoid tumor, also known as aggressive fibromatosis. It is a benign tumor, usually found in younger patients between 10 and 40 years of age, and is locally aggressive. It is often associated with pregnancy and previous surgery and can frequently recur. Management is en bloc, full-thickness, wide local excision (usually with frozen section confirmation of negative margins). An aggressive full-thickness abdominal wall resection is standard of care, making reconstruction more challenging.

The more durable and functional reconstruction entails complete restoration of the abdominal wall, especially musculofascial components, in a primary reapproximation. However, depending on the size of the tumor and resultant defect, this may not be possible. Basic principles, after obtaining proper margins after resection of the tumor, would then be reduction in defect size to the maximal extent possible and wide bridging underlay of mesh with at least 4- to 5-cm margins in all directions. Bridging inlay, whereby a mesh is simply sewn to the margins of the defect, has clearly been shown to be inferior in terms of recurrence rates.

Mesh choice can be either synthetic or biologic, although if significant contamination exists, if soft-tissue coverage is tenuous, or if one desires to decrease the amount of adhesion formation when placing mesh directly against the bowel, one should consider placement of biologic mesh, accepting the fact that there is a higher incidence of postoperative bulges using these materials, by and large.

In this case, only a left component separation is possible, given that the tumor has invaded the right rectus muscle and obliques, precluding their use for myofascial advancement.

There is no role for chemotherapy or radiation therapy in the treatment of these tumors.

Coverage with a right anterolateral thigh flap, with or without rectus femoris, can reconstruct the soft-tissue defect, but avoiding the use of mesh in a defect over 4 cm has a significantly higher chance of a recurrent hernia and would not be standard of care.

Primary skin grafting on top of a nonvascularized thick piece of acellular dermal matrix will not “take,” even with use of negative pressure wound therapy.

88
Q

A 57-year-old woman undergoes resection of a squamous cell carcinoma of the vagina, resulting in a defect of the posterior two thirds of the vaginal vault from the introitus to the dome. A small portion of the rectum is involved in the resection and primarily repaired. The patient has a history of pelvic radiation therapy and tobacco use. Which of the following is the most appropriate approach for closure of this defect?
A) Bilateral gracilis muscle flaps
B) Deep inferior epigastric artery perforator flap
C) Primary repair of the vaginectomy defect
D) Split-thickness skin grafting with obturator
E) Vertical rectus abdominis musculocutaneous flap

A

E) Vertical rectus abdominis musculocutaneous flap

In the clinical scenario described, the vertical rectus abdominis musculocutaneous (VRAM) flap is the most appropriate choice. This approach can provide enough tissue to resurface the vaginal vault and fill dead space.

For posterior defects of the vaginal vault, abdominal-based flaps are usually preferable. They can provide a large amount of vascularized tissue that rotates easily into the defect. Bilateral gracilis muscle flaps alone would offer less soft tissue, and mucosalization in a radiated field is unpredictable. A deep inferior epigastric artery perforator flap is far more complicated than a VRAM flap, and the donor site is problematic in patients with a history of smoking.

Primary closure of a large vaginal defect would likely cause significant stenosis and poor healing because of previous radiation. A skin graft with obturator is not likely to be successful given her radiation and tobacco history and violation of the rectum.

89
Q

A 62-year-old woman presents with a new-onset draining sinus of the left thoracic cage with associated indurated skin. Medical history includes bilateral breast cancer that was managed with bilateral radical mastectomy with radiation therapy 27 years ago. On CT scan, the image (shown) is consistent with osteoradionecrosis. Resection of affected skin, soft tissue, and thoracic cage produces a 35 × 20-cm soft-tissue defect and a skeletal defect spanning five ribs. A photograph of the defect is shown. The thoracic cage is fibrotic and noncompliant because of previous radiation. Which of the following approaches is most appropriate for reconstruction?

A) Free omental flap with skin graft over titanium mesh and reconstruction plates
B) Left latissimus dorsi muscle flap with skin graft over acellular dermal matrix
C) Left rectus abdominis turnover flap with skin graft over methyl methacrylate sandwich
D) Reverse abdominoplasty advancement over ePTFE patch
E) Right pectoralis muscle turnover flap over polypropylene mesh

A

B) Left latissimus dorsi muscle flap with skin graft over acellular dermal matrix

The most appropriate option for this patient is a left latissimus dorsi muscle flap with skin graft over acellular dermal matrix, given the alternatives listed. Basic principles of thoracic reconstruction include: debridement of devitalized tissue, removal of foreign bodies, establishment of healthy wound bed, restoration of stability/structure (generally reconstruction of skeleton if more than four ribs or a greater than 5-cm-diameter defect is involved), restoration of normal respiratory mechanics, protection of vital structures/organs, obliteration of dead space, provision of durable coverage, and delivery of an aesthetic result. However, if a patient has been previously irradiated, and therefore loses compliance of the thoracic cage because of radiation-induced fibrosis, skeletal reconstruction may not be mandatory if there is no paradoxical motion of the thoracic cage upon respirations and there is preservation of respiratory efficiency. Such is the case with this patient.

A left rectus turnover flap would not be a good option for two reasons: 1) as can be seen in the image, the left internal mammary artery has been harvested, thereby compromising the superior epigastric vessel on which this flap would be based, and 2) it is insufficient to provide enough soft-tissue coverage of a defect this size. Furthermore, as indicated above, methyl methacrylate would not be mandatory in this patient.

A right pectoralis turnover flap is insufficient to cover a defect this size.

A free omental flap can be used to reconstruct this defect (as can a pedicled omental flap), but again, thoracic skeletal reconstruction would not be mandatory in this previously irradiated patient; furthermore, even if it were, titanium mesh and reconstruction plates would not be utilized.

A reverse abdominoplasty flap (Ryan procedure) would not be able to cover a defect this size.

90
Q

A 56-year-old man is evaluated for a ventral hernia after undergoing midline laparotomy for diverticulitis. BMI is 38 kg/m2. Physical examination shows midline fascial defect measuring 20 × 15 cm; there is no evidence of infection and skin coverage is stable. Repair with rectus advancement and polypropylene mesh is planned. Placement of mesh between which of the following planes is most likely to decrease this patient’s risk of hernia recurrence?
A) Anterior rectus sheath and rectus muscle
B) Internal and external oblique muscles
C) Medial edges of rectus muscle
D) Rectus muscle and posterior sheath
E) Skin and anterior rectus sheath

A

D) Rectus muscle and posterior sheath

Although recurrence rates are generally very high for large ventral hernias, placement of mesh in the retrorectal position appears to have the most decreased rate of recurrence compared with other methods. Placement above the plane of the rectus muscle requires division of vascular perforators that traverse the rectus muscle and perfuse the overlying skin flaps. These perforating branches of the epigastric circulation are most dense in the periumbilical zone. Preservation of the perforators has been shown to be beneficial in a number of case series reports. Placement of mesh between the oblique muscles in this case would not provide support for the midline hernia because these muscles are more lateral.

Hernia recurrence rates are generally more increased in the presence of infection, with large defects when the rectus muscles cannot be advanced back together in the midline, in obese patients, and in patients with multiple medical comorbidities. Patients with several risk factors can expect recurrence rates in the range of 20 to 40%, whereas patients with few risk factors have recurrence about 5% of the time.

91
Q
A 60-year-old man undergoes sigmoid resection and colostomy for management of ruptured diverticulitis. The patient has smoked one pack of cigarettes daily for the past 35 years. BMI is 36 kg/m2. After colostomy reversal, he has an abdominal wound infection and fascial dehiscence. Reconstruction with a bridging human acellular dermal matrix is planned. Compared with traditional polypropylene mesh repair, which of the following complications is more likely with the planned approach?
A) Abdominal bulge
B) Fistula
C) Hematoma
D) Infection
E) Skin necrosis
A

A) Abdominal bulge

Acellular dermal matrices (ADM) have been advocated for the past decade as an important adjunct in the complex field of abdominal wall reconstruction. Many studies have verified the use and general safety of ADM in abdominal wall reconstruction, but conclusive evidence of its advantages over other techniques is still lacking. What can be inferred is its advantage over prosthetic mesh in contaminated fields. Polypropylene mesh would be contraindicated in the infected wound in this example. After adequate debridement and appropriate antibiotics in an optimized patient, ADM can be used for hernia repair or reconstruction along with component separation. Postoperative infections can be as common as 40%, but conservative management measures often suffice, rather than reoperation and graft explantation, which are required with prosthetic mesh.

One recognized drawback in the stretchable nature of dermal matrix grafts is that they can often stretch under tension to 50% or more of their initial dimensions. Postoperative bulging without true herniation is common. Strategies for prevention include suturing the graft under maximal stretch and use of porcine dermal grafts rather than human grafts.

Hernia recurrence in the complex abdominal wall reconstruction remains a common event, regardless of technique. Although some studies assert a strong advantage with ADM, others report a similar or increased recurrence rate. As successful operative techniques become more standardized, perhaps more uniform success will be demonstrated in future studies.

Skin necrosis and hematoma are common surgical complications that should not vary between choice of graft material.

Fistula rates are decreased with ADM versus prosthetic mesh reconstruction.

92
Q

A 40-year-old man undergoes ventral hernia repair with biologic mesh and fascial closure at the midline. A bilateral component separation technique with incision of the external oblique fascia and muscle lateral to the linea semilunaris and dissection in the plane between the external and internal oblique muscles, and separation of the rectus muscle off of the posterior rectus fascia is performed. At which of the following levels can the least amount of advancement of the medial fascial edges be expected?
A) Midway between the umbilicus and pubis
B) Midway between the umbilicus and subcostal margin
C) Subcostal margin
D) Suprapubic
E) Umbilicus

A

C) Subcostal margin

The component separation technique can be used to achieve medial transposition of the rectus muscle and overlying anterior fascia. The surgery involves division of the external oblique fascia and muscle lateral and parallel to the linea semilunaris. The plane deep to the external oblique muscle, which is relatively avascular, is then dissected laterally. The rectus muscle is also separated off of the posterior rectus sheath (using access from the medial laparotomy or hernia incision). This allows for medial advancement of the rectus muscle, overlying anterior rectus sheath, internal oblique muscle, and transversus muscle as a unit. The segmental neurovascular bundles course deep to the internal oblique muscle and penetrate into the rectus muscle 10 to 25 mm medial to its lateral margin.

The component separation technique, when performed in the scenario described, can give unilateral advancement toward the midline approximately 10 cm at the level of the umbilicus, which equates to a bilateral advancement of 20 cm. The least amount of advancement is in the subxiphoid and subcostal regions, often making more cranially located defects more difficult to close. Since it was originally reported in 1990, several modifications and variations of this technique have been described in the literature. These include perforator-preserving and/or endoscopic techniques to methods that describe additional maneuvers to increase mobilization or improve durability with the addition of biologic or prosthetic meshes.

93
Q
A 45-year-old woman comes to the office seeking consultation for an abdominoplasty. Physical examination shows diastasis recti and excess skin and fat in the infraumbilical region. Abdominoplasty with undermining of the upper abdominal flap and plication of the diastasis is planned. Decrease in overall sensation in which of the following areas is most likely after more than 3 years postoperatively?
A) Epigastric
B) Infraumbilical
C) Lateral abdominal
D) Subxiphoid
E) Suprapubic
A

B) Infraumbilical

The area below the umbilicus and above the incision is most likely to have decreased sensation in the long term (after more than 3 years). This is true for superficial touch, superficial pain, temperature (hot and cold), and vibration. The area above the umbilicus recovers sensation to touch, pain, and vibration within 3 years; sensation to hot and cold temperatures recovers beyond 3 years. The suprapubic area has decreased sensation to temperature; however, touch, pain, and vibration recover in the short term. The areas below the xiphoid, at the lateral abdomen, as well as all other areas on the abdomen, have decreased sensation to pressure, although the infraumbilical region had the greatest change.

The anterior nerve branches of the 6th to 12th intercostal nerves travel in a plane between the transversus abdominis and internal oblique muscles. They enter the internal oblique fascia, divide into two branches, and enter the posterior rectus sheath. One branch ascends 3 cm from the lateral edge of the rectus muscle to supply sensation to the skin over the lateral half of the rectus muscle. The other branch travels between the rectus and the posterior sheath before penetrating the linea alba and supplying sensation to the medial half of the rectus muscle. These nerves are divided during undermining from the abdominoplasty.

94
Q
A 65-year-old man is evaluated for a large mass in a previously irradiated area of the posterolateral chest. Preoperatively obtained ventilation-perfusion scans and pulmonary function testing are within normal limits. A full-thickness chest wall resection to include three ribs and at least 10 cm of each rib is planned. The soft-tissue defect is anticipated to be 20 cm in diameter. Which of the following factors in this patient is most likely to necessitate a rigid chest wall reconstruction?
A) Posterolateral position of the defect
B) Previous chest wall radiation therapy
C) Total area of the chest wall defect
D) Total number of ribs resected
A

C) Total area of the chest wall defect

The most likely factor to indicate rigid chest wall reconstruction in this very large anticipated chest wall defect is the total area of the bony chest wall that is resected. Generally, reconstruction of the lateral bony chest wall seems necessary if four or more consecutive ribs are resected or if the diameter of the total defect is larger than 5 cm.

Historically, protection of a flail segment of chest wall was based on the pendelluft principle, a phenomenon in which there is airflow back and forth between the lungs, resulting in underventilated segments of lung. This out-of-phase movement of the airway gas between the intact and flail-chest-side lungs was long believed to be the major contributor to respiratory dysfunction in patients with flail chest but has failed to be proven and appears to be a flawed hypothesis.

In patients who have had radiation therapy, larger defects may be tolerated without rigid chest wall stabilization owing to fibrosis. As many as five ribs may be resected in patients who have undergone radiation therapy before reconstruction is considered because increased fibrosis produces chest wall stiffness.

Anterior and posterior defects are typically better tolerated than lateral defects.

95
Q

Which of the following represents the lateral border of the breast footprint?
A) Anterior axillary line
B) Anterior edge of the latissimus dorsi muscle
C) Lateral clavicle
D) Midaxillary line
E) Posterior axillary line

A

A) Anterior axillary line

The lateral breast is mobile, unlike the inframammary fold. Therefore, the footprint can vary slightly and be just behind the anterior axillary fold, but it does not extend to the midaxillary line. The breast footprint is not described as relating to either the latissimus dorsi or the clavicle.

96
Q

A 43-year-old woman brings her 10-day-old daughter for evaluation because she is concerned about the appearance of her daughter’s nipples. The newborn was born at full-term after an uncomplicated pregnancy. Physical examination shows that the newborn has inverted nipples bilaterally. Which of the following is the most appropriate next step in management?
A) Continuous elastic outside distraction starting at day of life 14
B) Suction lengthening of the nipple starting at day of life 14
C) Suction lengthening of the nipple starting immediately
D) Transection of the fibrotic bands at 6 months of age
E) Observation

A

E) Observation

It is very common for nipples to be inverted at birth and concerned parents often discuss the condition with pediatricians and practitioners. The mammary ridge begins to develop in the fourth week of development, and produces the primary and secondary buds. The mammary pit is a small depression present at birth into which the lactiferous ducts open. After birth, proliferating mesoderm during the first several weeks of life will cause the inverted nipple to protrude.

In some cases, the inverted nipple persists. Although there may not be a functional consequence, it can lead to difficulty with lactation. Han and Hong described a grading system for inverted nipple: grade I, in which the nipple can be made to protrude manually and can maintain position without traction; grade II, in which the nipple fails to maintain projection after manipulation; and grade III, in which the nipple cannot be pulled out manually. Treatment is based on the grade. In some cases, breast-feeding or use of a breast pump is sufficient to correct the variation. Surgical intervention can be approached in several ways. Constricting the base of the inverted nipple can create projection, severing the ducts can cause projection, and myotomy can cause projection. Any division of the ducts will compromise a patient’s ability to breast-feed.

97
Q
Gynecomastia that occurs in men who take anabolic steroids is caused by increased levels of which of the following?
A) Aromatase
B) Estradiol
C) High-density lipoprotein
D) Testosterone
A

B) Estradiol

Anabolic steroids have sex-specific adverse effects. Development of breast tissue in men, a condition called gynecomastia (which is usually caused by high levels of circulating estradiol), may arise because of increased conversion of testosterone to estradiol by the enzyme aromatase. Decreased sexual function and temporary infertility can also occur in men.

98
Q

A 16-year-old girl comes to the clinic because she is dissatisfied with the asymmetric appearance of her breasts. Physical examination shows the right nipple-areola complex is more superiorly located and the breast volume is small. There is absence of the right anterior axillary fold. Which of the following other physical examination findings is most likely?
A) Accessory nipple
B) Microtia
C) Right clubfoot
D) Scars consistent with repaired cleft lip
E) Shortened right-hand digits

A

E) Shortened right-hand digits

The chest findings described are consistent with Poland syndrome with absence of the sternocostal head of the pectoralis major muscle. Poland syndrome can be associated with hand abnormalities, including shortened digits. Cleft lip, accessory nipple, clubfoot, and microtia are not known to be associated with Poland syndrome.

99
Q
A 19-year-old man with no available medical history presents for initial evaluation of gynecomastia. Which of the following is most appropriate to include in this patient’s workup?
A) Chest x-ray study
B) Hand x-ray study
C) Mammography
D) Testicular examination
A

D) Testicular examination

Gynecomastia can occur because of imbalances of estrogen and testosterone, and it can be associated with obesity, certain drugs, and Klinefelter syndrome. It can also be associated with testicular tumors. Therefore, a testicular examination should be performed. Mammography is not necessary for a routine workup if physical examination shows no abnormal masses. A chest x-ray study is not needed for an otherwise healthy 19-year-old. An x-ray study of the hand can evaluate closure of growth plates when determining if puberty has completed. This is unlikely to be necessary for someone who is 19 years old.

100
Q
The dominant vascular supply of the rectus abdominis muscle originates from which of the following vessels?
A) Common femoral
B) External iliac
C) Internal iliac
D) Internal mammary
E) Superficial femoral
A

B) External iliac

Component separation for closure of large abdominal wall defects was first described by Ramirez in 1990. The purpose of the surgery is to achieve abdominal wall closure with well-vascularized, innervated muscle flaps. The primary vascular supply to the rectus muscles are the deep inferior epigastric artery and vein, which arise from the external iliac vessels.

The internal mammary vessels give rise to the superior epigastric arteries and veins, which is a secondary, nondominant vascular supply of the rectus muscles. The femoral vessels give rise to the superficial inferior epigastric artery and vein, which perfuse the skin and subcutaneous fat of the inferior lateral abdomen.

101
Q
A 62-year-old man is evaluated for an 8-month history of fullness of the right breast associated with a subareolar mass. He reports no history of pain, nipple discharge, skin changes, or systemic symptoms. There is no family history of breast cancer. Examination of a specimen obtained on biopsy shows ductal epithelial hyperplasia with proliferation of stroma and fibroblasts. Which of the following is the most likely diagnosis?
A) Ductal carcinoma
B) Epidermal cyst
C) Fibroadenoma
D) Gynecomastia
E) Lymphangioma
A

D) Gynecomastia

Gynecomastia is defined as the benign proliferation of male glandular breast tissue. The most common symptom of gynecomastia is painless breast enlargement in adolescent and elderly men. Examination of a breast biopsy specimen may be necessary to exclude breast cancer if mammography and/or breast ultrasonography are suggestive of malignancy. Microscopic findings include ductal epithelial hyperplasia with proliferation of stroma and fibroblasts. Pseudogynecomastia is defined as breast enlargement secondary to fat deposition without glandular proliferation. Fibroadenoma are the most common benign tumor of the female breast. They are most often diagnosed in women between the ages of 20 and 35 years. Fibroadenomas are often ovoid or spherical, freely movable, and often well circumscribed. Microscopic findings include ductal tissue with proliferation of stroma surrounded by fibroblasts. Ductal carcinoma accounts for the majority of breast cancers. Ductal carcinoma is characterized microscopically by cords and nests of tumor cells with varying amounts of gland formation, and cytologic features that range from bland to highly malignant. Epidermal cysts are inclusion cysts lined by well-differentiated epidermis filled with keratin. Lymphangiomas of the breast are distended lymphatic channels interspersed with breast lobules.

102
Q

A 29-year-old woman comes to the office because of a firm, mildly tender, well-circumscribed mass of the abdomen. A photograph is shown. The mass has been slowly increasing in size for the past 6 months. CT scan shows a mass that occupies the left musculofascial abdominal wall, including the rectus, external, and internal oblique muscles, and penetrates through the anterior rectus sheath. Which of the following is the most appropriate management?
A) Neoadjuvant chemotherapy
B) Radiation therapy
C) Wide local excision, bilateral component separation, adjuvant chemotherapy
D) Wide local excision, mesh placement, radiation therapy
E) Wide local excision, right component separation, mesh reinforcement

A

E) Wide local excision, right component separation, mesh reinforcement

The lesion in this patient is a desmoid tumor, also known as aggressive fibromatosis. It is a benign tumor, usually found in younger patients between 10 and 40 years old, and is locally aggressive. It is oftentimes associated with pregnancy and prior surgery, and can frequently recur. Treatment is en bloc full-thickness wide local excision (usually with frozen section confirmation of negative margins). As aggressive full-thickness abdominal wall resection is standard of care, reconstruction is more challenging.
If midline fascia can be reapproximated, it should be, as primary fascial closure is associated with the lowest hernia recurrence rates. Reinforcement with mesh has been prospectively demonstrated to reduce recurrence rates even further, especially in defects over 4 cm. If midline fascial reapproximation is not possible, reduction in the size of the defect is crucial to decrease recurrence rates. This is done by component separation. However, in this case, only a right component separation is possible, given that the tumor has invaded the left rectus muscle and obliques, precluding their use for myofascial advancement. If the obliques had been spared, a component separation could still have been attempted even if there were violation of the rectus. The most durable reconstruction would be achieved if midline fascial reapproximation were possible with mesh reinforcement. Second best would be reduction in the size of the defect with a right component separation and placement of mesh as a bridging underlay.

There is no role for neoadjuvant chemotherapy or radiation therapy in the treatment of these tumors.

103
Q
A 21-year-old woman comes to the office because of a lump 2 cm below the inframammary fold. She says she has “always had it,” but it grew larger during pregnancy and has not decreased. The mass was painful during breast-feeding, and it is occasionally tender. Which of the following is the most likely diagnosis?
A) Epidermal inclusion cyst
B) Lipoma
C) Polymastia
D) Polythelia
E) Sarcoma
A

C) Polymastia

Patients often come to plastic surgeons with subcutaneous masses. An understanding of the differential diagnosis is helpful to counsel patients. Accessory breast tissue (polymastia) along the milk line is common. This breast tissue is responsive to hormonal influence, and patients will describe cyclical pain and swelling coinciding with menses and with milk letdown. Surgical excision can be performed. Polythelia is the presence of accessory nipples. Lipomas, epidermal inclusion cysts, and sarcomas can occur as subcutaneous masses, but they do not change in character based on hormonal influence.

104
Q

An otherwise healthy 27-year-old man comes to the office for removal of asymptomatic enlarged breasts that have persisted unchanged since onset at age 14. He takes no medications and does not use recreational drugs. Physical examination shows symmetrical collections of rubbery, firm subareolar tissue 4.5 cm in diameter. There is scant surrounding fatty tissue and no ptosis; areolas are 28 mm in diameter. Which of the following is the most appropriate management of this patient’s condition?
A) Circumferential periareolar resection with liposuction
B) Mastectomy via Wise pattern incision
C) Observation to allow for involution
D) Oral hormone-blocking medication
E) Subareolar tissue ressection via pull-through excision technique

A

E) Subareolar tissue ressection via pull-through excision technique

Gynecomastia, enlargement of the male breast, may occur as physiologic temporary overgrowth of the adolescent breast or it may appear during adulthood as a result of numerous etiologies. It can consist of various proportions of excess subareolar fibrous breast tissue and adipose tissue in the periphery, with the extent often depending upon the habitus of the individual. Appearing at about age 14 in more than 65% of healthy boys, gynecomastia will typically resolve within 2 years of onset in otherwise healthy males. Persistence of adolescent-onset gynecomastia after age 21 is unlikely to resolve with conservative measures. Hormone blockers have no place in the management of persistent breast tissue in otherwise healthy individuals. A wide variety of surgical treatment options for resection of redundant male breast tissue have been published. These include direct excision through a number of described incisions, traditional and ultrasound-assisted liposuction, and recently, use of an arthoscopic shaver. Ultrasound-assisted liposuction may have an advantage in stimulating skin retraction for cases where mild to moderate ptosis exists when scar minimization is desired. For otherwise uncomplicated gynecomastia, excision of fragmented subareolar tissue via a limited areolar border incision with the pull-through technique is the most appropriate choice of therapy. When redundant skin exists in addition to excess male breast tissue, immediate skin resection via periareolar, transverse lenticular, omega-shaped, or Wise (inferior pedicle) incision are among the numerous methods described. Regardless of the resection method, an evolving trend favors allowing skin to retract for 6 to 9 months before determining whether there is a need to subject the individual to additional incisional scarring.

105
Q
During fetal development, failure of the mammary pit to elevate above the skin level results in which of the following deformities?
A) Accessory nipple
B) Amastia
C) Amazia
D) Athelia
E) Nipple inversion
A

E) Nipple inversion

The congenitally inverted nipple is common and occurs in 4% of infants. It results from the failure of the mesenchyme to proliferate above the level of the skin.

Amastia is absence of the entire breast. Athelia is absence of the nipple. Amazia is absence of the mammary gland with an intact nipple and areola. Polythelia, or accessory nipple, results from failure of regression along the milk line.

106
Q

An otherwise healthy 14-year-old girl with Poland syndrome is brought to the office because of breast asymmetry and severe symptoms of right macromastia. She wears a size E cup brassiere and needs to fill the left side to attain symmetry in clothes. On examination, she has amazia and an absent sternal head of the pectoralis major muscle on the left side and macromastia of the right breast. Which of the following is the most appropriate reconstructive option for this patient?

A) Left-sided breast reconstruction with a latissimus dorsi flap and right-sided reduction mammaplasty
B) Left-sided breast reconstruction with a latissimus dorsi flap only
C) Placement of a subcutaneous left-sided tissue expander and right-sided reduction mammaplasty
D) No surgery until after puberty

A

C) Placement of a subcutaneous left-sided tissue expander and right-sided reduction mammaplasty

The most appropriate reconstructive option is to place a subcutaneous expander on the left side and perform a reduction of the right breast. This patient is still going through puberty and has a few years of growth left. However, her breast asymmetry is severe, symptomatic, and negatively impacting her life. Additionally, she has a good social support system with parents who are actively engaged in her well-being. Surgery that can potentially have permanent changes in a person’s physical anatomy (e.g., scars), and potentially negative consequences (e.g., inability to breast-feed) must be performed with very detailed and complete informed consent, especially in minors. On the other hand, doing nothing could also lead to potentially negative social, emotional, psychological, and physical consequences. Therefore, if the patient and parents understand the risks and benefits, and are fully informed, surgical treatment is warranted in situations such as the one described.

In this situation, simply placing an expander on the left side and not addressing the symptomatic right breast does not take care of all of her issues. A reduction of the right breast will not only help with her symptoms of breast hypertrophy, but will also enable a more symmetric reconstruction. Using a prosthesis to mimic an E cup breast is not ideal and will not succeed. Reducing the breast to about a C cup will allow the surgeon a realistic chance to reconstruct the left side to mimic the right breast.

The expansion process on the left side would then continue until volume symmetry is attained compared with the right side. Through her growing years, the expansion process could continue. When she stabilizes in terms of her breast growth, the expander can then be substituted with a permanent prosthesis. A contralateral further reduction/lift can also be performed as needed. An ipsilateral latissimus muscle flap can also be performed simultaneously to recreate the anterior axillary fold or fill the infraclavicular space if grossly concave.

However, the latissimus muscle flap in and of itself does not have the amount of tissue needed for larger breast reconstructions, as in this case, and needs a simultaneous expander placed under it for that reason.

107
Q

A 1-day-old female newborn is evaluated because of repair of a lumbar myelomeningocele. After dural repair, physical examination shows the spinal cord at the base of the wound with a 4 × 4-cm soft-tissue and skin defect. Which of the following is most appropriate to reconstruct the wound?

A) Gluteal muscle flap and skin advancement flap
B) Paraspinous musculofascial flap and skin advancement flap
C) Skin advancement flap only
D) Split-thickness skin grafting

A

B) Paraspinous musculofascial flap and skin advancement flap

The most appropriate method to reconstruct the wound is a local musculofascial flap and skin advancement flap. The major principle of myelomeningocele repair is to obtain a well-vascularized layer of soft-tissue coverage between the dural and skin repairs. The fascia overlying the paraspinous muscles can be turned over as flaps, followed by paraspinous muscle advancement flaps to cover the underlying dural repair. This vascularized soft-tissue layer will minimize the risk of cerebrospinal fluid contact with cutaneous bacteria and subsequent meningitis if either the dural repair or skin repair breaks down.

A split-thickness skin graft over the dura would not adequately protect the spinal cord. Closing the skin directly over the dural repair using skin advancement flaps would place the child at risk for meningitis in the event of a cerebrospinal fluid leak or if wound breakdown occurred along the incision line of the widely undermined skin flaps.
The use of a regional gluteal or latissimus muscle flap to cover the dural repair is unnecessary because local tissue (paraspinous muscles and fascia) is available. Harvesting the gluteal or latissimus muscles also may cause significant donor site morbidity in a child already at risk for ambulatory problems from a neurologic deficit.

108
Q

A 55-year-old woman with recurrent rectal cancer comes to the office for preoperative consultation for pelvic exenteration, which will include total vaginal resection and reconstruction with a vertical rectus musculocutaneous flap. Which of the following long-term complications is most likely in this patient?

A) Abdominal hernia
B) Pelvic abscess
C) Rectovaginal fistula
D) Small-bowel obstruction
E) Vaginal stenosis
A

E) Vaginal stenosis

Reconstruction of the vagina after oncologic resection can be challenging for plastic surgeons, as restoration of form and function must be considered. Restoration of form is often achievable by a variety of procedures, including the vertical rectus musculocutaneous flap, gracilis musculocutaneous flap, pudendal artery fasciocutaneous flap, skin grafting, and interpositional colonic grafts.

Restoration of functional outcome, however, is controversial. Return of sexual activity across multiple studies shows wide variability from 31 to 100% (53.8% pooled data). A recent survey study found that 50% of responders were sexually active and were able to achieve orgasm after reconstruction. However, a majority of responders reported wishing that they had been given more information on the procedure before surgery.

The most common long-term complication from total reconstruction of the vagina is stenosis with rates reported from 18 to 22%. This requires regular use of obturators for dilatation and this should be clearly discussed with patients preoperatively. Abdominal hernia, rectovaginal fistula, pelvic abscess, and small-bowel obstruction are less common complications.

109
Q

A 25-year-old man is scheduled to undergo soft-tissue coverage and nerve grafting using a seventh intercostal space nerve graft after he sustained a gunshot wound just above the left clavicle. On preoperative examination, he had difficulty abducting the shoulder, and injury to the brachial plexus was suspected. Between which of the following structures is the thoracic intercostal nerve located?

A) External intercostal and pectoralis major muscles
B) Innermost intercostal and internal intercostal muscles
C) Internal and external intercostal muscles
D) Parietal pleura and transverse thoracis muscle
E) Transverse thoracis innermost intercostal muscles

A

B) Innermost intercostal and internal intercostal muscles

Although not commonly used, the thoracic intercostal nerves can provide graft material up to 12 cm in length. Harvest of the intercostal nerve has minimal donor site numbness and can be harvested through an open or endoscopic approach.

The intrinsic chest muscles include (from superficial to deep) the external intercostal, internal intercostal, innermost intercostal, and transverse thoracis muscles. The external intercostal muscle is most active during inspiration. It functions to stiffen the chest wall during descent of the diaphragm to prevent paradoxical collapse of the chest. The other more internal intercostal muscles are weaker and are more involved in expiration.

The intercostal nerve emerges from the spinal cord and immediately splits into a dorsal ramus (that innervates the back) and ventral ramus. The ventral ramus runs between the internal and innermost internal intercostal muscles before crossing over the internal thoracic vessels and penetrating through the intercostal muscles to supply the anterior chest skin.

110
Q

A 32-year-old woman is scheduled to undergo subtotal reconstruction of the vagina using a pedicled deep inferior epigastric artery perforator flap after tumor ablation. Which of the following is the most likely advantage of using this type of flap coverage rather than the pudendal thigh (Singapore) flap?
A) Better maintenance of sensory innervation
B) Less vaginal bulkiness
C) Less vaginal hair growth
D) More optimal mucus secretion
E) Single-staged procedure

A

C) Less vaginal hair growth

Vaginal reconstruction for congenital vaginal agenesis or after tumor resection remains a challenging reconstructive endeavor. The ideal technique provides a vaginal canal with adequate dimensions, texture, appearance, and sexual function without excessive donor site morbidity.

The deep inferior epigastric artery perforator flap as a local regional flap has been described recently for vaginal reconstruction. It is a non-hair-bearing flap that is performed in a single stage but is insensate. Depending on the patient’s body habitus, it is less bulky than the musculocutaneous flaps (gracilis, vertical rectus) used for vaginal reconstruction, but more bulky than the Singapore flap. Two perforators are included in the flap to ensure viability. The major disadvantage is a conspicuous donor site scar.

The pudendal thigh flap, commonly referred to as the Singapore flap, is a local fasciocutaneous flap that has been widely used for neovaginal reconstruction. It is a thin, sensate flap based on the posterior labial arteries, which are a continuation of the perineal artery. Modifications of the flap have been described to enhance the viability and reach of the flap. It is a reliable, single-staged operation, but can result in endovaginal hair growth. This can be improved with preoperative electric or laser depilation of the vulvar portion of the flap.

Neither flap secretes mucus. Both are single-staged operations.

111
Q

A 27-year-old woman is scheduled to undergo female-to-male transgender surgery. A fasciocutaneous radial forearm flap is designed to construct the phallus. This procedure includes neurorrhaphy of the lateral antebrachial cutaneous nerve to a terminal branch of which of the following?

A) Genitofemoral nerve
B) Iliohypogastric nerve
C) Ilioinguinal nerve
D) Posterior femoral cutaneous nerve
E) Pudendal nerve
A

E) Pudendal nerve

In women, the pudendal nerve terminates in the dorsal nerve of the clitoris, which is responsible for most erogenous sensation. Neurorrhaphy to this nerve has been shown to result in superior sensory outcomes in phalloplasty. The dorsal nerve of the clitoris is analogous to the dorsal nerve of the penis, which is used as a target for neurorrhaphy during penile reconstruction for amputation or congenital microphallus. Other branches of the pudendal nerve include the inferior rectal and perineal nerves. The ilioinguinal nerve supplies sensation to the skin of the upper medial thigh and to the skin of the mons pubis and labia majora. The genitofemoral nerve also provides sensation to the labia majora and skin of the femoral triangle. The posterior femoral cutaneous nerve innervates the skin of the posterior surface of the thigh as well as part of the perineum. The iliohypogastric nerve innervates the skin above the pubis and in the gluteal region.

112
Q

To maintain innervated muscle flaps during abdominal component separation, which of the following is the most appropriate plane of dissection?

A) Below subcutaneous fat pad, above fascia
B) Below fascia, above external oblique
C) Below external oblique, above internal oblique
D) Below internal oblique, above transversalis
E) Below transversalis, above peritoneum

A

C) Below external oblique, above internal oblique

Component separation for closure of large abdominal wall defects was first described by Ramirez in 1990. The purpose of the surgery is to achieve abdominal wall closure with well-vascularized, innervated muscle flaps. The muscles of the abdominal wall are innervated by the intercostal nerves from T7-L4. The plane below the external oblique and above the internal oblique is an avascular plane that will allow for medial advancement of the rectus muscle flaps while protecting the intercostal nerves that run under the internal oblique.

113
Q

A 43-year-old woman with a history of ovarian cancer is evaluated because of recurrence in the incision and bladder dome. History includes resection, chemotherapy, and radiation therapy. The gynecologic surgeon plans a wide resection of skin, fascia, bilateral lower rectus muscles, and bladder dome resulting in a 10 × 16-cm defect in the central lower abdomen. Which of the following is the most appropriate procedure to achieve abdominal closure?

A) Bilateral component separation
B) Primary closure only
C) Primary closure with interpositional mesh
D) Use of a free latissimus muscle flap
E) Use of a pedicled anterolateral thigh flap

A

E) Use of a pedicled anterolateral thigh flap

The patient described will have a 10 × 16-cm, full-thickness defect of the abdominal wall in the setting of previous radiation. A full-thickness defect this large in the setting of past radiation makes primary closure alone and primary closure with interpositional mesh a poor choice. Bilateral component separation will not be possible because the rectus muscles will be resected with the specimen. A pedicled anterolateral thigh flap will offer well-vascularized, nonradiated tissue that will include soft-tissue coverage as well as fascia for abdominal wall reconstruction. A free latissimus muscle flap, while providing well-vascularized tissue, would involve the increased risks of microsurgery and not provide fascia for closure. Therefore, Option E is most appropriate.

114
Q

A 42-year-old man is brought to the emergency department after sustaining a traumatic, sharp amputation of the penis at the level of the proximal shaft during a domestic abuse incident. Repair of which of the following structures is necessary to obtain successful replantation of the penis?

A ) Cavernous artery
B ) Circumflex artery
C ) Deep dorsal artery
D ) Pudendal artery
E ) Subtunical venous plexus
A

C ) Deep dorsal artery

Before the advent of microsurgical techniques, the procedure to replant a penis was rudimentary – surgeons would simply suture the approximate corporal bodies and the overlying tunica. Although successful occasionally, this technique was prone to partial necrosis of the glans or shaft and was associated with sexual and erectile dysfunction. The use of microsurgical techniques has allowed surgeons to repair transected structures more precisely and achieve significantly improved results. A review of the current published literature on penile replantations finds that, in addition to repair of the urethra, the three neurovascular structures that are repaired consistently are the dorsal nerves, dorsal veins, and the deep dorsal artery. Generally, a suprapubic catheter is placed to divert urinary flow for 2 to 3 weeks while the urethral repair heals.

The cavernous artery is buried within the substance of the corpus cavernosum and is technically difficult to repair.

The circumflex artery runs circumferentially through collateral circulation and would not be repaired normally.

The pudendal artery is a branch of the internal iliac vessels and is located much more proximally in the pelvis. It gives rise to the inferior rectal artery, perineal artery, posterior scrotal artery, and the deep dorsal artery.

The subtunical venous plexus is extremely small and would be technically difficult to repair.

115
Q

A 32-year-old woman comes to the office for consultation regarding gender reassignment surgery. She is in a stable relationship with a supportive partner and has the support of her family. She is interested in mastectomy, hysterectomy, salpingo-oophorectomy, and phalloplasty. Referral to which of the following is the most appropriate next step in management?

A) Breast surgeon for mastectomy
B) Endocrinologist for hormonal manipulation
C) Gynecologist for hysterectomy and salpingo-oophorectomy
D) Psychiatrist for evaluation
E) Urologist to assist with phalloplasty

A

D) Psychiatrist for evaluation

The most appropriate next step in management is referral to a psychiatrist for evaluation. Gender identity disorder, formerly known as transsexualism, should ideally be managed by a multidisciplinary team. Ideal management includes psychotherapeutic, hormonal, and surgical treatments. Initial care involves a diagnostic phase in two stages. The goal of the first stage is to establish the diagnosis according to well-defined criteria and to rule out other psychiatric disorders. The second stage of the diagnostic phase is a ?real-life test,? in which the patient assumes the role of the opposite sex. Ongoing supportive psychotherapy is often required. Some centers begin hormonal therapy during the ?real-life test? to assist the transition and to make the test more realistic. Most centers defer hormonal therapy until 1 year following initial contact with the patient; surgery is deferred for 2 years following contact. There are no well-established protocols to guide the surgical sequence.

116
Q

A 1-day-old female newborn is evaluated because of a 4 × 4-cm defect after undergoing dural repair of a lumbar myelomeningocele. Which of the following is the most appropriate next step in management?

A) Negative pressure wound therapy
B) Split-thickness skin grafting
C) Coverage with a skin advancement flap
D) Coverage with a local musculofascial flap
E) Coverage with a free flap
A

D) Coverage with a local musculofascial flap

The most appropriate next step in management is coverage with a local musculofascial flap. The major principle of myelomeningocele repair is to obtain a well-vascularized layer of soft-tissue coverage between the dural repair and skin repairs. The fascia overlying the paraspinous muscles can be turned over as flaps followed by paraspinous muscle advancement flaps to cover the underlying dural repair. This vascularized soft-tissue layer will minimize the risk of contact of cerebrospinal fluid with cutaneous bacteria and subsequent meningitis if either the dural repair or skin repair breaks down.

Vacuum-assisted wound therapy over a dural repair would increase the risk of cerebrospinal fluid leak, dural injury, and breakdown of the repair. A split-thickness skin graft over the dura would not adequately protect the spinal cord. Closing the skin directly over the dural repair would place the patient at risk for meningitis in the event of a cerebrospinal fluid leak, or if wound breakdown occurred along the incision line of the widely undermined skin flaps. The use of a free flap to cover the dural repair is unnecessary because local tissue (paraspinous muscles and fascia) is available. Harvesting the gluteal muscle(s) may cause significant donor site morbidity in a child already at risk for ambulatory problems from a neurologic deficit.

117
Q

A 70-year-old man is evaluated because of chest wall incision drainage associated with leukocytosis, high fever, and blood cultures positive for Staphylococcus aureus 14 days after undergoing quintuple coronary artery bypass grafting using both internal mammary arteries. History includes type 2 diabetes mellitus, hypertension, hypercholesterolemia, and chronic obstructive pulmonary disease. He has smoked one pack of cigarettes daily for the past 40 years. BMI is 32 kg/m2. In addition to broad-spectrum antibiotic therapy, which of the following is the most appropriate initial management of this condition?

A) Debridement and negative pressure wound therapy
B) Debridement and reconstruction with a pectoralis turnover flap
C) Debridement and reconstruction with sternal plating
D) Debridement with primary rewiring and wound irrigation

A

A) Debridement and negative pressure wound therapy

Post-sternotomy mediastinitis is a severe complication, with an incidence ranging between 0.2 and 8%. Risk factors include advanced age, diabetes mellitus, morbid obesity, reoperation for bleeding, and use of bilateral internal mammary arteries. Deep sternal wound infections are more serious and have a higher mortality rate than superficial or sterile sternal wounds. The most common organisms are Staphylococcus aureus and Staphylococcus epidermidis, but mixed gram-positive and gram-negative infections are not uncommon. Broad-spectrum antibiotic therapy to include coverage of MRSA infection and Pseudomonas should be instituted. Nevertheless, debridement is the mainstay of therapy.

Adequate sternal and soft-tissue debridement is vital and can be combined with immediate sternal closure with delayed primary wound closure versus delayed closure of the entire wound with or without flaps to aid in obliterating any dead space. Many studies have demonstrated that the use of negative pressure wound therapy for wound coverage as a bridging method to final wound closure decreases the morbidity of these patients and is associated with decreased recurrent infection and treatment failure rates compared with conventional therapy, specifically primary rewiring or plating, especially for those at high risk.

Flap closure at the time of sternal debridement can be performed; however, if bilateral internal mammary arteries are used for the bypass, a pectoralis turnover flap is not an option because of loss of the internal mammary artery for the pectoralis major. Options would include coverage with the pectoralis transposition flap based on the thoracoacromial artery, an omental flap, or a rectus abdominis flap based on the intercostal artery.

118
Q

A 59-year-old man with hypertension, peripheral vascular disease, and coronary artery disease has sternal osteomyelitis after coronary artery bypass grafting with saphenous vein and left internal mammary artery grafts. Reconstruction with bilateral pectoralis advancement flaps is performed, but the flaps do not survive. Debridement is performed, and a defect remains. A photograph is shown (The defect encompasses the entirety of the sternum, from the sternal notch to the xiphoid.). Which of the following is the most appropriate reconstruction?

A ) Adjacent tissue transfer and coverage with bilateral skin advancement flaps
B ) Coverage with left rectus abdominis turnover flap and skin grafting
C ) Coverage with left superior epigastric artery perforator fasciocutaneous flap
D ) Coverage with omental flap and skin grafting
E ) Coverage with right latissimus myocutaneous flap

A

D ) Coverage with omental flap and skin grafting

The most appropriate option in the patient described is an omental flap with skin grafting. The defect encompasses the entirety of the sternum, from the sternal notch to the xiphoid. The first-line muscle flaps for this defect would usually be bilateral pectoralis muscle flaps (that do not include a left pectoralis turnover flap option due to the lack of a left internal mammary artery (IMA) used for the coronary graft). However, this option has already been used.

An omental flap has an axial blood supply that is not compromised (the gastroepiploic) and the ability to completely span the defect and obliterate the dead space; the skin graft provides the skin coverage needed to complete the reconstruction. Hence, it is the optimal choice in this patient.

An adjacent tissue transfer of random skin flaps bilaterally would not obliterate the dead space and would not provide robust axial blood supply to the necessary area, with the tenuous coverage existing along the sternal midline, exactly where the most vital blood supply would be necessary.

A left rectus abdominis turnover flap would not be an appropriate option for two reasons: 1) the left IMA has been harvested, thereby compromising the superior epigastric vessel on which this flap would be based; and 2) it would not reach the upper portion of the defect.

A left superior epigastric artery perforator fasciocutaneous flap would not be an appropriate option for two reasons: 1) the pedicle is compromised from the left IMA harvest; and 2) muscle flaps have been proven to be optimal for sternal defects more than fasciocutaneous flaps.

A right latissimus muscle flap may be a viable choice to get full coverage of the defect with dead space obliteration. However, the option indicates this is a myocutaneous flap, which would not allow for sufficient skin to be harvested to reconstruct the size of this defect.

119
Q

An otherwise healthy 55-year-old woman is diagnosed with squamous cell carcinoma of the vagina. Subtotal vaginectomy, vulvectomy, and inguinal lymph node dissection are performed. Bilateral sensate posterior thigh flaps are used for vaginal reconstruction. Which of the following nerves must be included in this procedure?

A ) Iliohypogastric
B ) Ilioinguinal
C ) Inferior gluteal
D ) Posterior femoral cutaneous
E ) Pudendal
A

D ) Posterior femoral cutaneous

The nerve that gives sensation to the posterior thigh flap is the posterior femoral cutaneous nerve (S1 to S3). This fasciocutaneous flap is based on the descending branch of the inferior gluteal artery and provides an abundant and reliable amount of available soft tissue for transfer from the posterior aspect of the thigh. Because of the proximal vascular pedicle, this flap can easily reach high within the pelvis and can be rotated to provide closure to both anterior and posterior vaginal defects. In addition, the flap can also be harvested with relative technical ease in a short period of time, and the donor site can typically be closed without the need of skin grafts. The skin is innervated by the posterior femoral cutaneous nerve (S1 to S3), which allows this flap to be used as a sensate flap when indicated. There are several other advantages to the posterior thigh flap that make it ideal for complex perineal reconstruction. The inferior gluteal artery is a terminal branch of the internal iliac artery.

120
Q

An otherwise healthy 17-year-old girl with congenital vaginal agenesis is referred to the office for reconstruction of the vagina. Physical examination shows normal external genitalia but an absent vaginal canal. She desires a single-stage procedure with minimal scarring and maximal sensation. Which of the following is the most appropriate reconstructive option for this patient?

A ) Bilateral gracilis musculocutaneous flaps
B ) Bilateral pudendal fasciocutaneous flaps
C ) Extended groin flap
D ) Split-thickness skin graft and mold (McIndoe procedure)
E ) Vertical rectus abdominis musculocutaneous flap

A

B ) Bilateral pudendal fasciocutaneous flaps

The most appropriate management is reconstruction with bilateral pudendal fasciocutaneous flaps. These flaps are based on the branches of the superficial perineal vessels, which are the terminal branches of the internal pudendal vessels. The course of the superficial perineal nerve follows these vessels; therefore, this flap can be raised as a sensate flap. These are relatively simple flaps to raise, and they are reliable and completed in one stage. They are thin, sensate flaps and are ideally suited for vaginal reconstruction. The donor site can be closed primarily and the scar is well hidden within the inner groin crease.

The McIndoe procedure involves a split-thickness skin graft reconstruction of the vagina using a mold, followed by the continued use of a vaginal dilator to prevent constriction. The skin graft donor site scar is not as inconspicuous as a linear scar in the groin crease. Extended groin flaps have been described for vaginal reconstruction; however, these flaps are insensate. Gracilis and rectus abdominis musculocutaneous flaps are workhorse flaps for vaginal and pelvic reconstruction. However, they too are insensate and have large, conspicuous donor site scars.

121
Q

A 4-year-old boy is scheduled to undergo abdominal wall repair using a component separation technique. He underwent living donor liver transplantation during infancy, at which time the graft was noted to be too large for size. A midline incisional hernia was intentionally made to facilitate organ function after transplantation. Current physical examination shows ventral herniation with marked loss of domain. During abdominal wall repair by component separation, parallel aponeurotomy is most likely to be performed immediately lateral to which of the following anatomical structures?

A ) Arcuate line of Douglas
B ) Inguinal ligament
C ) Linea semilunaris
D ) Midline
E ) Tendinous inscription
A

C ) Linea semilunaris

Separation of components of the abdominal wall is fast becoming a first-choice procedure for large ventral defects. The procedure involves medial advancement of the rectus abdominis muscle flap after division of the aponeurosis or fascia of the external oblique muscle layer.

Properly identifying the correct fascia to incise is a critical step in the procedure, and it relies on knowledge of the abdominal wall anatomy. The rectus abdominis muscles are separated by the midline linea alba, and each rectus muscle is medial to the layered triad of muscles—the external oblique, internal oblique, and transversus. Division of external oblique fascia is properly performed lateral to the linea semilunaris, which represents the lateral border of the rectus sheath, which is formed by the contributions of the external oblique, internal oblique, and transversus layers as the envelope of the rectus abdominis muscle. Above the arcuate line, the later layers split around the rectus muscle at the level of the internal oblique, forming both the anterior and posterior rectus sheaths; however, below the arcuate line, all layers travel anteriorly, forming the anterior sheath but no distinct posterior rectus sheath. The inguinal ligament exists between the anterior superior iliac spine and the pubic symphysis and should not be divided in component separation. Transverse tendinous inscriptions interrupt the rectus sheath, and division of the anterior rectus sheath or the inscriptions is not part of the classic component separation.

122
Q

A 45-year-old man undergoes abdominal reconstruction with a component separation technique. After release of the external oblique muscle bilaterally, which of the following best explains the preservation of flexion of the trunk?

A ) Preservation of the nerves between the anterior rectus fascia and pyramidalis muscle
B ) Preservation of the nerves between the external oblique and internal oblique muscles
C ) Preservation of the nerves between the internal oblique and transversalis muscles
D ) Preservation of the nerves between the transversus abdominis and the peritoneum

A

C ) Preservation of the nerves between the internal oblique and transversalis muscles

The rectus muscle is the primary flexor of the pelvis. Its segmental nerve supply enters the rectus sheath laterally in a plane between the internal oblique and transversalis muscles. Separation of components is a versatile and powerful technique for autologous abdominal wall reconstruction. Along with the ability to close large midline ventral defects primarily, the rectus muscle remains innervated because only the aponeurosis of the external oblique is released. If correctly performed, the internal oblique layer will shield the underlying nerve supply to the rectus and result in a functional abdominal wall repair.

123
Q

The blood supply to the mid-abdomen originates primarily from which of the following arcade regions?

A ) Deep epigastric
B ) Deep internal mammary
C ) Mid epigastric
D ) Superficial epigastric
E ) Superficial internal mammary
A

A ) Deep epigastric

Blood is supplied to the abdominal wall both by direct cutaneous vessels and musculocutaneous vessels. Haddad demonstrated the exact locations of these vessels using arteriography. Huger later classified the vascular supply into simple zones for use in abdominal lipectomy. This classification can be applied to abdominal reconstruction. The mid-abdomen is supplied primarily by the deep epigastric arcade. The superior epigastric artery arises from the internal thoracic artery and descends with the posterior rectus sheath. The deep inferior epigastric artery arises from the external iliac artery and ascends with the posterior rectus sheath. Each of these vessels supplies branches to the rectus muscle and overlying skin.

The lower abdomen is supplied by branches of the epigastric arcade and the external iliac artery. The superficial epigastric and superficial external pudendal arteries originate from the femoral artery running superficial to the fascia to supply the overlying skin. The inferior epigastric artery on the posterior rectus sheath supplies the underlying muscle in this zone. The deep circumflex iliac artery supplies the muscles in the area of the anterior iliac spine. The flanks and lateral abdomen are supplied by the intercostal, subcostal, and lumbar arteries. These vessels arise from the aorta and give off perforators to the back and flank. They travel circumferentially on the transversus abdominis, perforating the oblique muscles and supplying the overlying skin.

124
Q

An otherwise healthy 35-year-old man comes to the office because of a 15-year history of bilateral Grade III gynecomastia. Height is 5 ft 10 in (178 cm) and weight is 187 lb (85 kg). BMI is 27 kg/m2. Physical examination shows breast enlargement with skin redundancy and palpable glandular and fatty tissue. Ultrasound-assisted lipectomy is planned because the patient wishes to avoid visible scars. Which of the following is most likely to minimize the need for skin resection?

A ) Disruption of the inframammary fold
B ) High energy setting
C ) Peripheral feathering with standard liposuction
D ) Treatment of the intermediate fat layer
E ) Tumescent infiltration with a 3:1 ratio

A

A ) Disruption of the inframammary fold

Although surgery remains the mainstay of treatment for gynecomastia in this country, results have been inconsistent, with reported dissatisfaction rates as high as 50%. Common aesthetic problems reported after surgery include unacceptable scarring and nipple and areolar deformities such as tethering and malposition. As a result, less invasive techniques have evolved and are now the mainstay of treatment for all but the most severe forms of gynecomastia.

The use of ultrasound-assisted lipectomy has improved the outcomes in the treatment of all grades of gynecomastia. In the more severe grades, certain maneuvers will improve skin retraction and redraping and may thus eliminate the need for a subsequent procedure for skin excision. While fatty and fibrous tissue is best approached from a deeper subcutaneous plane, transition to a subdermal plane can allow for greater skin retraction. Moreover, disruption of the inframammary fold is essential in that it allows the skin to drape more naturally onto the abdomen, which is typical of a male breast.

The use of wetting solutions is imperative with ultrasound-assisted lipectomy to avoid burns. Both superwet (1:1) and tumescent (3:1) techniques are acceptable. The power of the unit should generally not exceed 90% to avoid burns and dermal injuries. Peripheral feathering improves contour, providing a smoother transition to the outer border of the breast, but does not specifically address skin excess.

125
Q

Which of the following is the most common hyperplastic childhood breast anomaly?

A ) Giant fibroadenoma
B ) Gynecomastia
C ) Juvenile hypertrophy
D ) Polymastia
E ) Polythelia
A

B ) Gynecomastia

Breast anomalies in children are a relatively common finding. A recent study showed that hyperplastic abnormalities were the most common category, with the most common anomalies being gynecomastia, followed by juvenile hypertrophy or hyperplasia. The average age at initial surgery for the group was age 17.4 years; the average number of operations per patient was 1.14. Patients most likely to require revisional surgery were those with giant fibroadenoma.

126
Q

Which of the following best represents the likelihood of malignancy in adolescents undergoing subcutaneous mastectomy for gynecomastia?

A ) 1%
B ) 5%
C ) 10%
D ) 15%

A

A ) 1%

A literature search yielded over 2000 articles in total; however, only 36 articles have discussed cases of adolescent gynecomastia and the associated pathologic results, resulting in data for 615 individuals. Of these 615 individuals, there have been six cases of cancer and five cases of atypical ductal hyperplasia associated with adolescent gynecomastia. Specific patient information was only available for seven of the individuals (six breast cancer and one atypical ductal hyperplasia), which revealed that the average age of patients involved was age 17.4 years (range, age 16 to 20 years), 43% of cases had symptoms of unilateral gynecomastia, and an abnormal physical examination was present in only one case. In the cases with histologic characterization, five cases were ductal carcinoma in situ with low to intermediate grades, and the other case had been diagnosed as invasive carcinoma of the secretory carcinoma type. Sixty percent of the cases of ductal carcinoma in situ had associated atypical ductal hyperplasia as well.